From 00276fc59c20520731464c556c5d874407ae60cf Mon Sep 17 00:00:00 2001 From: "W. Trevor King" Date: Fri, 20 Apr 2012 17:07:16 -0400 Subject: [PATCH] Fix typos in Serway and Jewett v8's 23.62 and 25.21. --- latex/problems/Serway_and_Jewett_8/problem23.62.tex | 2 +- latex/problems/Serway_and_Jewett_8/problem25.21.tex | 4 ++-- 2 files changed, 3 insertions(+), 3 deletions(-) diff --git a/latex/problems/Serway_and_Jewett_8/problem23.62.tex b/latex/problems/Serway_and_Jewett_8/problem23.62.tex index 6c9dc06..eb338d3 100644 --- a/latex/problems/Serway_and_Jewett_8/problem23.62.tex +++ b/latex/problems/Serway_and_Jewett_8/problem23.62.tex @@ -1,6 +1,6 @@ \begin{problem*}{23.62} Four identical charged particles ($q=+10.0\U{$\mu$C}$) are located on -the corners of a rectangle as shown in Figrue P23.62. The dimensions +the corners of a rectangle as shown in Figure P23.62. The dimensions of the rectangle are $L=60.0\U{cm}$ and $W=15.0\U{cm}$. Calculate \Part{a} the magnitude and \Part{b} the direction of the total electric force exerted on the charge at the lower left corner by diff --git a/latex/problems/Serway_and_Jewett_8/problem25.21.tex b/latex/problems/Serway_and_Jewett_8/problem25.21.tex index 403a08f..116769b 100644 --- a/latex/problems/Serway_and_Jewett_8/problem25.21.tex +++ b/latex/problems/Serway_and_Jewett_8/problem25.21.tex @@ -1,10 +1,10 @@ \begin{problem*}{25.21} -Two particles each with charge $+2.00\U{$\mu$C}$ are lopcated on the +Two particles each with charge $+2.00\U{$\mu$C}$ are located on the $x$ axis. One is at $x=1.00\U{m}$, and the other is at $x=-1.00\U{m}$. \Part{a} Determine the electric potential on the $y$ axis at $y=0.500\U{m}$. \Part{b} Calculate the change in electric potential energy of the system as a third charged particle of -$-3.00\U{$\mu$C}$ is brought in from infinitely far away to a positino +$-3.00\U{$\mu$C}$ is brought in from infinitely far away to a position on the $y$ axis at $y=0.500\U{m}$. \end{problem*} -- 2.26.2